Legal Ethics Test

¡Supera tus tareas y exámenes ahora con Quizwiz!

Charles in an attorney in Athens, Alabama and is the only attorney in the area that specializes in medical malpractice claims. Charles specializes in the area because his brother was a victim of a terrible medical malpractice case that ended his brother's life and resulted in his parent's divorce. A doctor in Athens is accused of experimenting with his patients without their permission to obtain a patent on a medical device. The case has received excessive press coverage internationally. The doctor went bankrupt defending himself against criminal charges arising from the case and was practicing without medical malpractice insurance. The doctor is unable to find an attorney that will take the case pro bono and has no one to represent him. What must Charles do? 1. He must represent the doctor but only if can overcome his repugnance for the client. 2. He must represent the doctor because all attorney must take their "fair share" of unpopular cases. 3. He must take the case because he is the only qualified attorney in the area and attorneys are required to do 50 hours of pro bono work a year. 4. Charles does not have to take the case.

. Charles does not have to take the case.

Jenny Smith and Maggie Brown graduate from law school and pass the Bar. However, the economy and over-supply of attorneys in their community make it difficult to obtain employment. They decide to share rent in an office space close to the courthouse and get on the appointed attorney list for criminal defense. They do not form a partnership because they realize the conflict of interest rules would restrict their practices. However, they share a secretary and split the cost of office supplies. Finally, they place a shingle over their door that reads: "Smith & Brown, Attorneys at Law" and has a little pair of pink boxing gloves in the corner and the phrase, "Fight like a GIRL!" Are the attorneys subject to discipline? 1. Yes, advertising that suggests bias toward or against women is prohibited in all states. 2. Yes, if the shingle implies they are in a partnership. 3. No, as long as the secretary informs everyone who enters or calls that Smith and Brown have separate practices. 4. No, attorneys are allowed to share office space, secretarial staff and office supplies.

. Yes, if the shingle implies they are in a partnership

While working on a terrible personal injury case against a famous movie star, an associate followed a hunch that some of the star's fans must have cell phone footage of the car wreck. After months of working with the star and her fan base, the associate was finally able to piece together enough cell-phone footage to show that the accident was not the star's fault. The movie star was so grateful to the associate that she asked him, "What is your biggest dream in life?" The associate, a workaholic, told her he had always dreamed of opening a boutique law firm in Manhattan. The movie star told the associate she would lend him $1 million dollars to open his own law firm. She offered him generous repayment terms, basing his monthly repayment amount as 25% of the net profit from the month before. In exchange, she asked the associate to represent her for free on any future cases and give her 2% of any future recoveries over $1 million dollars. Will the associate be subject to discipline if he takes the offer after the client obtains the advice of outside counsel? 1. Yes, the loan itself might be acceptable, but requiring a percentage of future recoveries is not. 2. Yes, the loan itself might be acceptable, but basing repayment on net profits and requiring a percentage of future recoveries in not. 3. No, attorneys are not limited in what loans they obtain or how they repay the loan and investing in a law firm does not create a partnership. 4. No, unless the investment puts pressure on the attorney to take cases the star approves of and so interferes with his professional judgment.

. Yes, the loan itself might be acceptable, but requiring a percentage of future recoveries is not.

A defense attorney is about to finish up a long week when an old client of his walks in his office. He hands his secretary a box and asks her to "hold onto it for him" and leaves. The secretary tells the attorney what the client did. The attorney inquires as to what the client means, and the secretary says she heard the client's ex-wife was found dead this morning. How must the attorney ethically respond? 1. He should give the box to his secretary and tell her to "get rid of it." 2. He shouldn't ask the client what is in the box and stick it in his office somewhere for safe keeping. 3. He should inform the client that he will have to give the District Attorney any evidence in his possession. 4. He should examine the contents of the box so he will know how to defend his client and then deposit it in a safe place. The attorney-client privilege protects the information.

3. He should inform the client that he will have to give the District Attorney any evidence in his possession.

An attorney battling pneumonia negligently advises his client to plead guilty in a DUI case where the arresting officer has moved to Russia and would never be available to testify. Realizing his dreadful mistake after his head began to clear from his illness, the attorney notified his client in writing he should seek independent counsel about whether to sue him for malpractice. A few months later, the attorney called the client and learned he never got around to looking for an attorney. The attorney offered the client a sizeable settlement to keep the mistake quiet and the client gladly agreed. Is the attorney subject to discipline? 1. Yes, he should have called the Bar and notified them of his malpractice. 2. Yes, he entered into a settlement with the client when the client didn't have an attorney. 3. No, attorneys can settle malpractice claims outside of court. 4. No, he told the client to get independent counsel in writing and it was his client's choice not to do so.

4. No, he told the client to get independent counsel in writing and it was his client's choice not to do so.

A group of atheists hires a lawyer to sue a school to obtain preliminary and permanent injunctions against the school for their use of a "moment of silence" before the beginning of the school day. The lawyer is the nation's leading expert on First Amendment rights and charges $500 per hour for her services. The group agrees to pay the rate. They give her $50,000 in advance for attorney fees and $10,000 for future litigation expenses. The lawyer properly deposits the entire amount in her IOLTA account. The lawyer spends 100 hours preparing the complaint, arguing and making motions, and appealing the case and spends $5,000 in court costs. After losing her appeal to the 5th Circuit, the lawyer suggests they appeal to the Supreme Court but the atheists are furious the lawyer has lost every motion and appeal. They fire her and decide to hire a different lawyer. The demand she refund the entire $60,000 and hand all paperwork to the new attorney. Which of the following must the lawyer immediately give to the former client? 1. $60,000 and all the documents she filed and opposing counsel filed but not her work product. 2. $60,000 and all the documents. 3. None of the documents as they are public records and $5,000. 4. All of the documents and $5,000.

All of the documents and $5,000.

A plaintiff comes to Lawyer Luke and explains that he is very unhappy with his current lawyer and wishes for Lawyer Luke to take the case. If the plaintiff wins the case, Luke would make several million dollars. However, in the midst of their engagement interview, plaintiff admits that during the defense's depositions he lied about the existence of a pre-existing back injury he got when he was in high school playing football. The injury is material to the case but very hard for the defense to prove because a fire destroyed the medical records of the plaintiff years ago. Plaintiff tells the lawyer he plans on testifying at trial exactly as he did in the deposition. What is proper for the attorney to do in this situation? 1. Decline to represent the client. 2. Agree to represent the client and keep the client's confidential information secret even if he testifies as planned. 3. Decline to represent the client and inform the court that the client lied in his deposition. 4. Agree to represent the client but tell him when he lies on the stand the judge will be informed he is lying.

Decline to represent the client.

A defense attorney is working on a terrible family violence case. The victim does not want to prosecute but suffered substantial, documented injuries. The defendant has a history of abusive conduct toward his partners. The morning of the trial, the victim does not appear in court. The judge promised to dismiss the case if the victim does not appear in court the last time the case was set. Just before the case is called, the defense attorney overhears his client boast to his brother that, "I scared that tramp stiff last night. No way is she coming to court." What is the proper course of conduct? 1. Withdraw and allow the judge to appoint a new attorney to oversee dismissal of the case. 2. Inform the client that he could be prosecuted for witness tampering but maintain silence as it is client confidential information of past criminal actions. 3. Disclose the client's actions to the court. 4. Do nothing. The client did not perjure himself.

Disclose the client's actions to the court.

A police officer was charged with savagely beating and ultimately killing a teenage gang member in the course of an arrest. Neither the police department nor the officer's union was willing to provide legal counsel for his defense because of the clear abuse of his authority, and the officer himself lacked funds to hire private counsel. The trial court appointed an attorney with three years of experience in criminal defense. The attorney has never tried a murder case. For which of the following reasons may the lawyer decline the court appointment? 1. Based on what he has read in the newspapers, the attorney believes that the officer is guilty. 2. He represented the victim on a Possession of Marijuana case last year. 3. He is the same race as the victim and has sympathy for the plight of teenagers caught up in gangs. 4. He has never tried a murder case.

He represented the victim on a Possession of Marijuana case last year.

A federal judge in Mississippi was assigned a case involving protesters who were arrested at an abortion clinic for impeding traffic into the facility. Prior to the start of the hearing, the judge announced from the bench that he needed to disclose that his wife volunteered at the Sav-A-Life Pregnancy Center both counseling women against abortion and providing economic assistance to enable women to maintain their pregnancies. Both sides agreed to waive their right to disqualify the judge as impartial for that issue. As the case proceeded, the judge realized that one of the protesters who was arrested was a distant cousin. Next, he suddenly recognized that an attorney sitting at the counsel table with the prosecution was his former law-clerk from a decade or more ago. The judge remembered the law-clerk to have been exceptionally bright and as soon as he saw him felt that the prosecution's arguments must be legally sound. Now the judge began to feel uncomfortable about ruling on such a contentious issue and all the publicity that might surround his ruling. He stopped the attorneys mid-argument and said, "Counsel, I'm afraid I've made a mistake. I have strong, personal beliefs that Roe v. Wade was incorrectly decided and as such I do not believe that I could be impartial. I've decided to disqualify myself." Which of the following propositions is true? 1. The judge did the right thing disqualifying himself based on the controversial nature of the case. 2. The fact the judge's cousin was arrested means he must disqualify himself. 3. If the judge has a personal bias toward his former law clerk he must disqualify himself. 4. The judge's wife's volunteer work at Sav-A-Life is not a waivable bias.

If the judge has a personal bias toward his former law clerk he must disqualify himself.

Google owns thousands of software copyrights. The company uses a large firm in San Jose, CA to handle all of its legal work. The company granted several software licenses to small start-up businesses in Austin, TX. None of the firm's attorneys representing Google are licensed in Texas for some strange reason. The software license agreement was drafted by Google's attorneys but contains an arbitration clause that mandates any disputes take place in the more business-friendly state of Texas. Google's attorneys flew to Texas to negotiate the software licensing contract. Unfortunately, soon after the start-up businesses began utilizing the software, Google discovered the start-ups were utilizing Google software copyrights outside the scope of the agreement. Pursuant to the arbitration clause, Google sent the firm's best litigator to Texas to arbitrate the dispute. Assume both jurisdiction have adopted all applicable ABA Model Rules. Which of the following is the best answer? 1. It was proper for Google's attorneys to draft the agreement, including the arbitration clause. It was proper for Google's attorneys to represent Google at both the negotiation stage of the agreement as well as at the arbitration in Texas. 2. It was proper for Google's attorneys to draft the agreement, including the arbitration clause. It was proper for Google's attorneys to represent Google at the negotiation stage of the agreement. 3. It was proper for Google's attorneys to draft the agreement, including the arbitration clause. 4. It was not proper for Google's attorneys to practice law in Texas.

It was proper for Google's attorneys to draft the agreement, including the arbitration clause. It was proper for Google's attorneys to represent Google at both the negotiation stage of the agreement as well as at the arbitration in Texas.

A sitting federal judge who was being interviewed by a newspaper reporter said, "He is a faker," of the presumptive Republican presidential nominee. "He has no consistency about him. He says whatever comes into his head at the moment. He really has an ego. ... How has he gotten away with not turning over his tax returns? The press seems to be very gentle with him on that... I can't imagine what this place would be -- I can't imagine what the country would be -- with Donald Trump as our president." Assume that the judge is bound by the ABA Canons of Judicial Conduct. Are the judge's comments proper? 1. No, judges must be circumspect and neutral about all issues. 2. No, a judge may not publicly endorse or oppose any candidate. 3. Yes, a judge may oppose publicly other candidates when running for re-election. 4. Yes, judges maintain the First Amendment right to Free Speech.

No, a judge may not publicly endorse or oppose any candidate

In a high profile civil trial, the plaintiff's attorney hired an expert in reading non-verbal communication to help him pick the jury and read the juries reactions to the testimony. The expert sat with the attorney at the counsel table and offered input and advice after every stage. The expert recommended settling the case while the jury was deliberating because he believed they would find that the defendant was not negligent when they returned with a verdict. The attorney took the expert's advice and made an offer that the defendant agreed to pay. After the agreement was signed, the judge brought the jury back and told them the case had settled after all. The jury said they had just agreed to find for the plaintiff with a much larger damages award than the settlement that was reached. Is the attorney subject to discipline? 1. Yes, the expert was a third party that the attorney allowed to sit at the counsel table. 2. Yes, the expert directed and regulated the legal services of the plaintiff. 3. Yes, an expert may be paid to render advice on issues in a trial but not the ultimate result of the trial. 4. No, as long as the attorney did not give up his discretion to the expert.

No, as long as the attorney did not give up his discretion to the expert.

Attorney B's niece was murdered by her significant other in a family violence case. Attorney B had a crisis of conscience as she often defended criminals who beat their significant other and she was beginning to feel very differently about those cases. In fact, because Attorney B was a fabulous trial attorney she had helped many clients escape any consequences for their violent behavior. Even though Attorney B still represented dozens of people accused of Domestic Violence, Attorney B became a member of the legal services branch of a local abuse shelter so she could help victims of abuse find jobs and obtain a divorce. One of the victims of a client she represents is living at the abuse shelter with her children. Is Attorney B subject to discipline? 1. Yes, the attorney cannot help an organization that could serve persons having interests adverse to her client. 2. Yes, because the victim of her client is living at that abuse shelter. 3. No, as long as the attorney is not knowingly participating in taking an action that would have a materially adverse affect on her client or require confidential information of said client be divulged. 4. No, even if Attorney B's clients are repugnant to her she should not hesitate to represent them.

No, as long as the attorney is not knowingly participating in taking an action that would have a materially adverse affect on her client or require confidential information of said client be divulged

Linda practices family law. One of her current clients is having a hard time making ends meet (or find employment) during the pendency of her divorce because the client has been out of the workforce for years. The client advertises on Facebook that she is starting a Mary Kay business. Linda knows her client needs the money and wants to help her get the business off its feet so she places a large order for Mary Kay make-up and sundries. Is Linda subject to discipline? 1. Yes, a lawyer shall not enter into a business transaction with a client without informed consent in writing. 2. Yes, a lawyer shall not provide financial assistance to a client. 3. No, as long as the goods are routinely marketed to the public by the client. 4. No, the gift is not in connection to pending litigation

No, as long as the goods are routinely marketed to the public by the client.

A real estate attorney wants to provide a full range of services to her clients. She enters into an arrangement with a licensed real estate broker and registered insurance agent to refer one another cases. Each will practice their own profession separately but they will refer clients to each other non-exclusively. For example, if the real estate broker has a home buyer who needs a real estate attorney she will refer the buyer to the lawyer. If the lawyer has a client who needs life insurance, she will refer the insurance agent. All clients and customers are informed of the arrangement. Is the attorney subject to discipline? 1. Yes, a lawyer shall not give anything of value to a person for recommending the lawyer's services. 2. Yes, reciprocal referral agreements must be between lawyers not non-lawyer professionals. 3. No, the reciprocal agreement is non-exclusive and clients are informed of the agreement. 4. No, as long as they also agreed to limit the referrals for a definite duration and the agreement never interferes with the attorney's professional judgement.

No, as long as they also agreed to limit the referrals for a definite duration and the agreement never interferes with the attorney's professional judgement.

A woman visits Attorney B whom she hires to represent her in a will contest. The client tells you she hired Attorney A at first but it's been months and he hasn't communicated with her or filed any papers. She said she went to his office last week since he hadn't answered her calls and he was intoxicated and mean. She asked her husband to go visit Attorney A but he was treated the same way and wanted her to find another attorney. She says she sent a certified letter to Attorney A notifying him that if he didn't respond in three days he could consider himself discharged. It has been two weeks since she sent the letter. Attorney B realizes that the will contest has a very short statute of limitations and papers must be filed immediately to be within the statute. Attorney B calls Attorney A to get information about from the client's file. Attorney A has no idea he had been discharged and told Attorney A his wife was diagnosed with terminal cancer two months ago and he has not been able to cope as he is unable to pay her medical bills. He asks the attorney to keep his situation confidential but tell the client of his wife's diagnosis and beg for her forgiveness. Attorney A was able to timely file the will contest and, under the circumstances, decided not to report Attorney B to the disciplinary authorities. Were the attorney's actions proper? 1. Yes, because the attorney is bound by the duty of confidentiality. 2. Yes, because the client gets to decide whether to report Attorney A or not. 3. No, because Attorney A's actions indicate he is not fit to practice law and thus both Attorney B and A are subject to discipline. 4. No, because Attorney B must tell his client to report Attorney A.

No, because Attorney A's actions indicate he is not fit to practice law and thus both Attorney B and A are subject to discipline.

Tom is a tax attorney who exclusively defends individuals and entities against delinquency and tax evasion claims by the IRS. Although he has a large and thriving practice, he wishes to expand his clientele. However, most people who have problems with the IRS are embarrassed and wish to keep the matter private. Tom knows, because of his familiarity with the public record system in the courthouse, how to identify people with tax matters pending. So, every month, he scours the records for email addresses and emails potential clients the following: "Advertising Material: Do you have BIG-TIME tax problems? I have specialized in defending individuals and entities against tax delinquency and evasion claims by the IRS for many years. My website has free articles answering certain tax issues and other relevant information. Visit: www.tomthetaxman.com or reply to this email to set up a free consultation. Advertising Material." Is Tom subject to discipline for his e-mail? 1. No, because public records are an acceptable place to look for new clients and attempt to make contact by email even if they have made it known to the attorney they do not want to be contacted. 2. No, because a lawyers may solicit clients by email and may communicate the fact the lawyer does or does not practice in particular fields of law. 3. Yes, because a lawyer may not state or imply that he or she is certified as a specialist in a particular field of law unless he or she has been certified as a specialist and the organization is approved by the ABA or relevant state bar authority. 4. Yes, because he offered free legal advice to the general public as a method of attracting clients and without carefully evaluating their particular scenarios.

No, because a lawyers may solicit clients by email and may communicate the fact the lawyer does or does not practice in particular fields of law.

A young associate in a criminal defense firm assists a partner in preparing for trial in a terrible murder trial. The defendant insists on testifying and the two lawyers woodshed him before trial to help prepare him for cross-examination by the prosecution. The defendant plans on testifying that he was happily married, loved his wife and never intended to harm her. He wants to tell the jury that the sleeping pills he took before bed the night of the murder caused him to hallucinate and brutally kill his wife. The young associate believes that the defendant did not love his wife and was not happily married—in fact, he desperately wanted out of the marriage and had gone to see a divorce attorney a few months before the murder. The associate believes that testifying that he loved his wife and was happily married is perjury and that they should tell him not to testify this way. However, the older partner, who is also the associate's supervisor, disagrees. He does not believe it is perjury to claim he loved his wife because of the fickle nature of relationships. He will not let the associate advise the client not to testify that he loved his wife and was happily married or tell the court that the client intends to perjure himself. May the associate inform the judge after the testimony that the client perjured himself? 1. No, because the associate should abide by the partner's resolution of an arguable matter. 2. No, the associate must keep his client's confidences. 3. Yes, because the associate had a duty to bring perjury to the court's attention. 4. Yes, because the associate must not allow another person to interfere with his professional judgment

No, because the associate should abide by the partner's resolution of an arguable matter.

An attorney working on a personal injury case met with the opposing counsel at a mandatory settlement conference. The opposing counsel offered the best settlement offer yet. It was more than the attorney had dreamed the case could win at trial. The attorney labored for hours trying to attempt to get his client to take the settlement. However, the client insisted on rejecting the settlement offer and demanding a jury trial. The attorney knew that the case rested on complicated legal understanding that a jury would have difficulty grasping much less finding for the client. So, although the attorney begrudgingly turned down the settlement offer he waived jury and set the case for a bench trial. The client was furious because she believed jurors were less jaded than judges. Is the attorney subject to discipline? 1. No, because the attorney may waive jury in civil cases. 2. No, both settlement and waiving jury are part of legal strategy. 3. Yes, because both the settlement and whether to waive jury was up to the client and not the attorney to decide. 4. Yes, because jurors are less jaded than judges.

No, because the attorney may waive jury in civil cases

A famous attorney who is also a CPA starts his own accounting firm. The attorney offers financial planning and tax related services. Because the business only offers accounting work and not ancillary legal services, the clients are not advised that their rights in regards to confidentiality are not the same as if it were a law firm. The firm does not protect client confidentiality in the legal sense when performing accounting services. However, most of the clients are aware that the attorney is in charge of the business. Is the attorney's conduct proper? 1. No, an attorney may not provide both law and accounting services in the same business venture. 2. No, because the attorney should have made sure clients knew the rules of professional conduct do not apply to the accounting business. 3. Yes, the ABA Rules apply to legal work not accounting work. 4. Yes, because accounting is not a law-related service

No, because the attorney should have made sure clients knew the rules of professional conduct do not apply to the accounting business

A conscientious defense attorney takes a child molestation case because he honestly believes that the defendant is wrongfully accused. The defendant is accused of molesting his ex-girlfriend's children and the attorney believes that she made up the accusations to get back at the defendant for leaving her for another woman. The attorney puts his client on the stand as well as his mother and they both testify that at the time of the alleged assault his mother was helping him watch the children and he was never alone with them and could not have committed the atrocious acts that are alleged. The defendant is acquitted of the charges. While celebrating the verdict, the defendant's mother lets is slip that "she was so afraid they would find those pictures on the internet." The defense attorney goes home with a sick feeling in his stomach. Overcome with guilt, he looks on-line with facial recognition software for the child victims and finds terrible pictures documenting the abuse that defendant was just acquitted of perpetrating. Must the defense attorney reveal the perjury? 1. No, because the disclosure violates the duty of confidentiality and the case is closed and cannot be appealed for double jeopardy reasons. 2. Yes, the defense attorney must reveal the mother's perjury but not the defendant's. 3. Yes, the defense attorney must reveal the defendant committed perjury. 4. Yes, the defense attorney must reveal both the defendant and his mother's perjury.

No, because the disclosure violates the duty of confidentiality and the case is closed and cannot be appealed for double jeopardy reasons.

A lawyer represents a defendant in a terrible serial murder case. The defendant's mental faculties are the only issue that may save his life but the lawyer is not sure where to start in finding expert's to evaluate his mental fitness. The lawyer's former law professor is a nationally recognized expert on death penalty cases, but is not licensed in this state. The lawyer calls his old professor and asks for help—and to keep the conversation confidential. To frame the issue accurately, the lawyer reveals to the professor information the client gave him in confidence. However, he never tells the professor his client's name. Is the lawyer subject to discipline for disclosing the client's confidential information to the professor? 1. Yes, because the professor was not licensed in that jurisdiction. 2. Yes, because the client did not consent to the disclosure. 3. No, because the disclosure was necessary to effectively carry out the representation of the client. 4. No, because as long as you don't reveal a client's name you can always talk about client confidences.

No, because the disclosure was necessary to effectively carry out the representation of the client.

A lawyer is a member of the Climate Change Alliance (CCA), an organization devoted to using the courts to prevent activities that cause harm by contributing to global warming. The lawyer holds a town hall meeting in a community that abuts the property where a coal-fired power plant is planned to be built. After extolling the danger of pollution at the meeting, the lawyer approaches a mother with her infant and asks her to allow her to sue the power plant on her behalf to stop the construction. After walking up to talk with her, the lawyer realizes she knew the mother in grade school. She truthfully tells the mother that she will represent her for free and all expenses will be paid by the CCA. However, the lawyer is not actually that interested in climate change. She thinks the case will get a lot of publicity and she will gain a reputation as a trial attorney in the community. Is the lawyer subject to discipline? 1. Yes, the lawyer engaged in face-to-face solicitation which is prohibited conduct for attorneys under the Model Rules. 2. Yes, because the lawyer is seeking a pecuniary gain from all the clients she expects to retain after working on a high profile case. 3. No, because the lawyer did not solicit the local resident with a motive of pecuniary gain. 4. No, because the lawyer knew the mother before she solicited her as a client.

No, because the lawyer did not solicit the local resident with a motive of pecuniary gain.

Attorney Alex had attended church at First Baptist of Birmingham his entire life. One of his old Sunday school teachers called him and asked him to help her with some trusts and estates work. She wanted him to write her a will that the residue of her estate would go to the church after the proceeds from the sale of her significant portfolio was distributed to her heirs. Alex suggested his partner serve as executor and worked hard to ensure all the paperwork was as it should be. A few years later, the client died and her estate was distributed. One of the client's children approached Alex at church and stuck an envelope in his hand. When he opened the envelope, it was a check for $20,000 from the deceased client with a note that read, "My mind has been at rest that all the details would be worked out after I die because of you. Please take this gift." Is the attorney subject to discipline? 1. Yes, attorneys may not receive substantial gifts and they are not related. 2. Yes, attorneys may not write a will and have their partner serve as executor. 3. No, because there is no evidence that the client lacked capacity. 4. No, because the unsolicited gift was not written in the will and there was no undue influence.

No, because the unsolicited gift was not written in the will and there was no undue influence

A lawyer seeks to increase her business. She is approached by Lawyer's Internet Referral Service, Inc. (LIRSI) For a fee, LIRSI will add the lawyer to a roster of lawyers listed on their web site. If anyone clicks on her name and visits her website, lawyer will pay LIRSI a set amount. The lawyer decides it is an easy way to attract more clients and signs herself up for the service. The service works and she begins to see a steady influx of new clients. At the same time, her neighbor approaches her about setting up a Trust for her children. The neighbor is so pleased with her work that she recommends lawyer to several of her friends who also come to lawyer for Trust and Estate issues. Lawyer sends her neighbor a bouquet of flowers with a note thanking her for her recommendation. Is the lawyer subject to discipline? 1. Yes, because the ABA model rules forbid lawyers from being paid for referrals. 2. Yes, she gave a gift to her neighbor who recommended her services. 3. No, but only if LIRSI is a qualified lawyer referral service approved by the state agency. 4. No, because she is not soliciting business. The clients are contacting her.

No, but only if LIRSI is a qualified lawyer referral service approved by the state agency.

In a hotly contested divorce, the matter of child custody has cost the parties tens of thousands of dollars. Finally, the husband's divorce attorney approached his client and suggested he meet with his wife in person and just "try to work it out" once and for all. The client called his wife and made an offer for joint custody on the condition they agree to a non-relocation clause. The ex-wife agreed. The father called his attorney who was over-joyed that his client had talked with the opposing party and worked it out since he was not ethically allowed to communicate with the represented party but always suspected her attorney was aggravating their issues. Is the divorce attorney subject to discipline? 1. Yes, an attorney may not get someone to do something he could not ethically do. 2. Yes, although his client could meet with the opposing counsel, he could not suggest it. 3. No, client's may meet without their attorneys and work out their problems even if they are both represented. 4. No, the divorce attorney may even meet with the ex-wife himself and talk about the case if they have agreed to a compromise.

No, client's may meet without their attorneys and work out their problems even if they are both represented.

Carlos asks his attorney to prepare a title search report for a potential buyer on a tract of land Carlos recently inherited from his grandmother. However, when the attorney looked into the title, she discovered that his grandmother had executed a quick claim deed to get the title out of probate after her husband had passed away. The fact it was a quick claim deed is not good for Carlos and may make the buyer back out of the purchase; but, considering no one else has a claim to the property the deed will likely prove to still be good title. The attorney goes ahead and sends a report to the buyer that the title search "is good" but has a quick claim deed issue. The buyer relies on the attorney's opinion that the deed should be fine and buys the house. Two weeks after selling the property the buyer learns a different relative owns the property. Was the attorney's conduct proper? 1. No, he needed informed consent to give the buyer the information about the quick claim deed and he may be liable to the buyer if the report was negligently done. 2. No, he needed informed consent to give the buyer the information about the quick claim deed. 3. No, attorneys cannot communicate with non-parties about client confidences. 4. Yes, the attorney should comply with reasonable requests by clients.

No, he needed informed consent to give the buyer the information about the quick claim deed and he may be liable to the buyer if the report was negligently done.

A presidential candidate holds a large rally that thousands of people attend. Although many of the people are supporters, there is a significant contingent of protestors. With all the raucous, one of the candidate's privately held security personal misconceives a protestor's cell phone case as a gun. The security personal assaults the protestor in an effort to apprehend him. The presidential candidate is sued by the protestor for negligent hiring and supervision of the employee. The candidate calls Attorney Angelica and asks her to handle the matter as quickly and cheaply as possible. Angelica warns the client that there were thousands of people at the rally and she really needs to conduct significant interviews to competently do the job. The candidate says, "This case is crap. I've got lots of experience with this sort of case and there is no way a jury will ever fine me a cent. Don't interview more than 5 people. Push this case to trial quickly so the press will see I'm vindicated." Unfortunately for the client, the jury pool was fairly poisoned against him and delivered a large verdict for the plaintiff. Afterwards, a supporter came forward with cell phone footage that showed the protestor was intentionally baiting the security personal and attempting to mislead security into thinking he had a gun. No jury would have ever found him at fault had they seen the footage. The candidate is furious with Angelica and sues her for malpractice. Is the attorney subject to liability? 1. No, if the attorney's actions were reasonable under the circumstances. 2. No, she was following the client's instructions and warned him of the possible repercussions. 3. Yes, a competent attorney would have insisted on interviewing the eye-witnesses to the assault. 4. Yes, the attorney had a duty to interview witnesses, she breached that duty and it caused her client significant harm.

No, if the attorney's actions were reasonable under the circumstances

Molly, a licensed attorney who practices criminal law in Texas, has a brother named Collin who lives in Tennessee. One day in July, her brother, a recent law school graduate, called in a panic. He got in an accident on the way to take the Tennessee Bar Exam. As the doors to the exam would close in 20 minutes, Collin begged to know whether he could simply leave his insurance information with the other driver and leave the scene of the accident. Molly said, "Collin, you know I am not licensed in Tennessee! I have no idea what the laws there look like. Here in Texas, you could leave the scene as long as no one is injured and you left your insurance information. Honestly, most states follow that rule. But, that could be entirely different in Tennessee. I would do it—but, without research it is risky." Collin left his information and went to the Tennessee Bar Exam. Unfortunately, in Tennessee you must stay at the scene of a collision until the police clear you to leave if there is property damage. Collin was cited. Is Molly subject to discipline? 1. Yes, even though Collin is her brother, she is giving him legal advice that she is not competent to give and she knows he will rely on that advice. 2. Yes, because she is not licensed in Tennessee and not competent. 3. No, if the court rules the matter was reasonable related to her practice in Texas and it was an emergency. 4. No, because she correctly stated the law in Texas.

No, if the court rules the matter was reasonable related to her practice in Texas and it was an emergency

An attorney is appointed to a "jail call" case where defendants too poor to afford attorneys have someone appointed and the state tries to make a deal with the defendant to move the case quickly through the system since the defendants cannot afford bail. As soon as the attorney gets the defendant's file from the state he realizes that the state ran the wrong criminal history. His client has already told him this is his fourth time to be arrested for drinking and driving. However, the criminal history the prosecutor's office ran does not indicate he has any criminal history because the prosecutor's staff failed to correctly input the client's driver's license number. The lawyer counsels his client to plead guilty quickly before they find the mistake. When they approach the bench, the judge asks the prosecutor if the defendant has any criminal history. The prosecutor says that this is his first offense. The judge finds the defendant guilty of a first offense DUI and sentences the defendant to back time. The statutory minimum sentence for a felony DUI (more than three priors) is five years in prison. The defendant and his attorney say nothing and take the sentence proffered. Is the attorney subject to discipline? 1. Yes, because failure to correct the judge when you know he is operating on false information is an affirmative misrepresentation. 2. Yes, because the judge asked for the client's criminal history. 3. No, even if asked directly by the judge, the attorney may not reveal confidential information. 4. No, if the mistake did not originate with the attorney or the accused.

No, if the mistake did not originate with the attorney or the accused.

An attorney working for the Texas Attorney General in their White Collar Crime and Public Corruption Unit helped prosecute numerous companies for Ponzi schemes. The attorney recently left the Texas Attorney General's office to work in a personal injury firm. A potential client approaches the attorney's new firm and explains she lost her entire retirement in a Ponzi scheme and would like to sue the corporation. The attorney's new firm realizes the client was injured by a particularly heinous corporation their new attorney personally prosecuted for the exact Ponzi scheme that injured the potential client. May the firm properly accept the case? 1. Yes, the new client's interests are aligned with the Texas Attorney General so there is no conflict of interest. 2. Yes, but only if the corporation consents to the firm representing the client in writing. 3. No, if the new attorney obtained confidential government information while prosecuting the corporation the government is prohibited from disclosing by law unless the new attorney is timely screened and apportioned no part of the fee. 4. No, unless the Texas Attorney General gives informed consent in writing.

No, if the new attorney obtained confidential government information while prosecuting the corporation the government is prohibited from disclosing by law unless the new attorney is timely screened and apportioned no part of the fee.

A famous personal injury attorney is always in high demand. He has his associates handle most of his cases and simply tries the cases that make it to trial. After losing a well-published case, the number of his cases going to trial have multiplied exponentially. The attorney has cases in dockets all across his jurisdiction and is having trouble keeping up with all the trial settings. Plaintiff Penny has a case pending with very little money in dispute. The attorney only took the case as a favor to Penny's mother. As such, the attorney has taken continuances eight times in favor of other cases with higher earning potential. Penny has called the attorney multiple times to ask when the case will actually be tried because she keeps taking off of work to be there when the case is ultimately postponed. She has never received a return phone call. Finally, the attorney shows up to court. As it turns out, he knew the defense attorney in law school and after talking amicably they agree to go out for lunch while the jury deliberates. Penny wins her case. Was the attorney's conduct proper? 1. No, it is the attorney's duty to zealously represent his client not fraternize with the opposing counsel. Also, it is his responsibility to return the client's phone calls, not take on more cases than he can handle, and not reset the case unreasonably. 2. No, it is his responsibility to return the client's phone calls, not take on more cases than he can handle, and not reset the case unreasonably for his personal convenience. 3. No, the attorney cannot simply try the cases, he must see them every step through from the beginning to the end. Moreover, it is his responsibility to return the client's phone calls, not take on more cases than he can handle, and not reset the case unreasonably for his personal convenience. 4. Yes, the attorney won the case and the plaintiff had the benefit of a renowned attorney.

No, it is his responsibility to return the client's phone calls, not take on more cases than he can handle, and not reset the case unreasonably for his personal convenience.

Charlotte, Lawyer Linda's client, has been indicted by the grand jury for failing to protect her 5 year old daughter from her boyfriend who molested the little girl. Linda strongly suspects that Charlotte's ex-husband was the actual perpetrator and the little girl accused Charlotte's boyfriend out of perverse familial loyalty. The prosecutor has offered to settle the case with only a few months of jail time at the little girl's request. Lawyer Linda is afraid that the little girl will be further molested by the ex-husband if he gains custody as a result of the plea. As such, Linda chooses not to communicate the plea offer to Charlotte and instead arranges for Charlotte and her daughter to undergo hypnosis from a licensed counselor with the hope that under hypnosis they will both lose their fear of the ex-husband and tell the truth about who is the perpetrator. Linda knows that due to a mandatory reporting statute the counselor will be duty bound to report whoever they implicate as the perpetrator. However, since Charlotte and her daughter have such a strange attachment and fearful loyalty to the ex-husband, Linda chooses not to mention the mandatory reporting statute to her client who would likely not go. Were the lawyer's actions proper? 1. Yes, because as an officer of the court the lawyer has a duty to protect the little girl. 2. Yes, not mentioning the mandatory reporting statute is fine. 3. No, although the lawyer did not have to mention the mandatory reporting statute to her client, she failed to communicate a plea offer to her client. 4. No, she should have told the client about the mandatory reporting statute as if it only benefited her daughter and she should have communicated the plea offer.

No, she should have told the client about the mandatory reporting statute as if it only benefited her daughter and she should have communicated the plea offer.

A lawyer runs a television commercial. He appears in an expensive suit which he otherwise never wears, and is surrounded by law books which he does not own or use since he does all his legal research online. In the advertisement, the attorney says: "My name is John Adams. I'm a licensed attorney. I believe my fees are fair and very low. In addition, almost all my clients are satisfied with my work. I am a member of the Alabama and U.S. Supreme Court bars." In fact, his fees are low, and based on client questionnaires, most of his clients are pleased with his work. Moreover, he is admitted in both Alabama and the U.S. Supreme Court. Does the advertisement violate the applicable ABA standards? 1. Yes, he cannot mention his clients without their express permission. 2. Yes, he is misleading the public. 3. No, the advertisement was truthful and not misleading. 4. No, because advertisement is protected under the First Amendment Commercial Speech doctrine.

No, the advertisement was truthful and not misleading.

After decades of practicing law, Burger, one of the named partners in the large partnership, MacDonald & Burger, died. Due to all the time he spent in the office, he was never able to sustain a marriage for longer than a few years. However, he did produce one heir, an undisciplined hippie who lived on the beach. Under the pre-existing partnership agreement, the heir will receive the capital contributions Burger contributed when the partnership first began, which liquidated from firm assets, amounts to about $10 million dollars. Also, the heir will receive another $100,000 in attorney fees Burger had earned but the firm had failed to collect from clients at his death. Last, the heir will receive a death benefit of $50,000 dollars, which represents a percentage of Burger's earnings the year prior to his death. The firm plans to pay all $10,150,000 in 10 monthly installments of $1,015,000. Does the plan subject the partnership to discipline if executed? 1. No, the firm may properly pay the heir all 10,150,000. 2. Yes, the firm may only pay the heir the $10 million capital contribution. 3. Yes, the firm may only pay the heir the $10 million capital contribution and the $100,000 in uncollected attorney fees Burger had already earned. 4. Yes, the firm may only collect the death benefit.

No, the firm may properly pay the heir all 10,150,000

A criminal court judge is approached by an attorney who frequently tries cases in front of the judge. The attorney offers to be the chair person for his re-election committee. As she is an excellent attorney, the judge entrusts her with the position. The attorney begins calling other attorneys who appear before the judge and soliciting campaign contributions. She puts together a list of attorneys who "endorse" the judge and publishes the list on a website she set up for the judge. Finally, she sets up an interview with a local newspaper about the judge's qualifications so that the paper could endorse him as a candidate. The paper sends a questionnaire for the judge to fill out prior to the interview regarding his views on the death penalty and victim rights. The judge complies and does the interview which results in the paper's endorsement of the judge. Under the CJC, were all actions proper? 1. No, judge's cannot pressure campaign contributions from those who may appear before them through an election committee. 2. No, judge's cannot pressure those who may appear before them to "endorse" them and publish the list. 3. No, judge's cannot seek partisan endorsements. 4. No, the judge filled out the questionnaire without explanation

No, the judge filled out the questionnaire without explanation

Donnie Dee finally finished 60 hours of law school classes and obtained his third-year bar card. He was able to find employment as a clerk with a local divorce attorney. Unfortunately, in the jurisdiction Donnie lives, a third year bar card does not allow students to practice law under the supervision of a licensed attorney with any more freedom than a paralegal. Still, the divorce attorney is over-loaded with cases and lets Donnie do whatever he is comfortable doing. Which of Donnie's actions could subject the divorce attorney to discipline for assisting in the unauthorized practice of law? 1. Drafting a divorce settlement using several templates that the divorce attorney has on file that the attorney reviews before asking her client to sign. 2. Interviewing neighbors and family members about cases involving spousal abuse and having them sign Donnie's written statement of their interview. 3. Reaching settlement agreements between spouses who both agree to the divorce and come to the attorney's office to get a divorce. 4. Undertaking extensive research and drafting memoranda.

Reaching settlement agreements between spouses who both agree to the divorce and come to the attorney's office to get a divorce.

Susie Que is a recent college graduate interning at local law firm to try to decide if she wants to go to law school. One day, on the way to work, she witnesses a terrible hit and run. She jumps out of her car and calls an ambulance for the victim. The police were able to apprehend the driver of the other vehicle (after the intoxicated driver ran into a median.) However, the paramedics could not arrive quickly enough to save the victim of the hit and run. Susie Que was there when the victim's wife arrived at the scene. The wife asked Susie how it happened and Susie told her what she had witnessed. Feeling terrible for the young stay-at-home mother, she offered her the firm's card and said, "My bosses are the best personal injury attorneys in town—if you come by our office, I'll make sure they take your case." Susie Que finally got to the office later that afternoon. First, she ran into Aaron, an associate in the firm, and told him what happened and what she told the young mother. "Good job!" he said. "I can't wait to stick it to the creep who was running around drunk as Cooter Brown at 8:00 am on a school day." Next, she ran into the managing partner and shared her crazy morning. The partner said, "Susie! Don't you remember when you started here I told you that you couldn't solicit customers to the firm? Never do that again." Later that month, the young mother called the associate and the associate took the case. Who is subject to discipline? 1. The associate because he took the case and told her "Good job!" 2. The partner because he didn't emphasize Susie Que's training enough for her to remember not to solicit clients in-person. 3. The associate for taking the case and the partner for failing to supervise Susie Que. 4. The associate for telling her "Good Job" and the partner for allowing the associate to take the case.

The associate for telling her "Good Job" and the partner for allowing the associate to take the case.

A famous actress claims that her director sexual assaulted her during production of a movie. She meets with attorney Z as a prospective client to discuss suing the director. After several lengthy discussions, the actress decides she wants a more 'high profile' attorney and hires someone else. The civil case is put on hold when the director is charged criminally. The case goes to trial and is televised. The attorney decides to go to the courthouse and watch the case live. The actress dramatically takes the stand and testifies about the assault. She testifies that they had "never been more than friends" before the assault. The attorney knows from his prior conversations with her that she and the director had an illicit affair many years before the assault while she was still married. What must the attorney do? 1. Inform the judge because the defendant will suffer substantial bodily harm if convicted. 2. Inform the judge because the victim committed perjury. 3. The attorney may break confidence if the defendant will suffer substantial bodily harm but he does not have to even though the victim perjured herself. 4. Inform the prosecutor that the victim perjured herself and let her take remedial action.

The attorney may break confidence if the defendant will suffer substantial bodily harm but he does not have to even though the victim perjured herself.

An attorney who normally practices civil law was appointed by the court to defend a client for a horrific murder. As the attorney was accustomed to civil cases, she sent her paralegal to start interviewing potential witnesses as soon as she was appointed. When her paralegal interviewed the client's elderly, reclusive neighbor, the neighbor told the paralegal that on the night of the murder, she looked out the window and saw the client drag a black bag into his trunk. She said she found it peculiar because the next morning was trash pick-up yet he loaded up the black bag and drove off with it. The victim's body had in fact been found in a black bag a few miles from the client's house. The neighbor died before the trial without ever speaking to the state authorities. Which of the following best states what the attorney should do with respect to the information she has learned from the landlord? 1. The attorney should voluntarily reveal the information to the prosecutor prior to trial because the death of the neighbor has made it impossible for the prosecutor to obtain the information another way. 2. The attorney is not obligated to reveal evidence collected by her paralegal 3. The attorney should keep the information in confidence unless the client authorizes her to reveal it, even though the death of the neighbor has made it impossible for the prosecutor to obtain the information in any way other than from the attorney. 4. The attorney should use her best judgment about how to treat the information; it is neither privileged nor confidential because her client didn't tell her the information

The attorney should keep the information in confidence unless the client authorizes her to reveal it, even though the death of the neighbor has made it impossible for the prosecutor to obtain the information in any way other than from the attorney.

In a tightly contested judicial election, a lawyer who practices election law represented a candidate who, several days before the election, told the lawyer in confidence that he had intentionally failed to report some large donations from attorneys who would stand to benefit from his rulings if elected. The lawyer was horrified as he had filed the paperwork himself and advised the client to report the donations immediately. As the candidate refused, the lawyer noisily withdrew from representing him but did not directly tell anyone what had happened. On Election Day, the candidate won by a narrow margin. A few days later, the truth about the donations came out and a front-page newspaper article accused the candidate and the attorney of conspiring together to mislead the general public. Which of the following propositions is false? 1. It was proper for the lawyer to withdraw after the candidate refused to correct the paperwork. 2. It was proper for the lawyer not to tell anyone what the candidate did if no 3rd parties were owed a duty. 3. After the newspaper story and a press conference, but before any formal proceeding, the lawyer may disclose what the candidate told him and that the attorney withdrew as soon as he learned of the failure to report the donations. 4. The attorney should wait until formal proceedings to break his duty of confidentiality and only reveal information that will not harm his former client.

The attorney should wait until formal proceedings to break his duty of confidentiality and only reveal information that will not harm his former client.

In the supplemental reading assigned for class today, which of these statements is false? 1. The attorneys in the Buried Bodies Case did not tell the media about where they found the bodies of the dead teens. 2. The attorneys in the Buried Bodies Case were ostracized by the community and suffered financial consequences as well as death threats. 3. The attorneys in the Buried Bodies Case never broke confidentiality with their client even when he escaped from prison. 4. The attorneys in the Buried Bodies Case had a fist fight with each other sometime after their client outed the fact the attorneys had known all along where the bodies were buried when the client took the stand.

The attorneys in the Buried Bodies Case never broke confidentiality with their client even when he escaped from prison.

A prospective client comes to an attorney's office for a free consultation without notice. The client explains that he is being sued civilly for aggravated assault. He explains that he sometimes uses cocaine and while high he got in an altercation with someone who cut him off at a red light. The client admits he has serious road rage issues. This triggers the attorney's memory and she suddenly realizes this must be the hot-head husband of the wife his partner is representing in a custody dispute. The attorney stops the client and tells him she cannot represent him since her partner represents his wife. Which of the following is true? 1. The partner must withdraw from representing the wife because the attorney received confidential information from the client that would be harmful to the client in the custody dispute. 2. The partner can represent the wife and the attorney can represent the husband because the two matters are not substantially related. 3. The partner may continue to represent the wife only if the lawyer who met with the prospective client is timely screened off from participation in the case, receives no part in the fee, and gives written notice of the situation to all parties. 4. The partner may continue to represent the wife if she gives informed, written consent.

The partner may continue to represent the wife only if the lawyer who met with the prospective client is timely screened off from participation in the case, receives no part in the fee, and gives written notice of the situation to all parties.

Maggie was appointed on a terrible intoxication manslaughter case. During her opening argument to the jury, Maggie said "The evidence will show my client is disabled and could never have performed the field sobriety tests. I have personally observed my client unable to walk from one side of my office to the other many times. No one can pass those tests! In fact, the prosecutor in this room failed the field sobriety tests when trying to demonstrate the tests to a different jury. This case is a witch hunt out for innocent blood and you must find my client not-guilty. " In fact, Maggie's client was disabled but the evidence of the disability was not admissible because Maggie had failed to timely file the medical records or get an affidavit from the custodian of the records at the hospital. Which statement was not a violation of the ABA rules? 1. "This case is a witch hunt out for innocent blood and you must find my client not-guilty." 2. "The evidence will show my client is disabled and could never have performed the field sobriety tests." 3. "The prosecutor in this room failed the field sobriety tests when trying to demonstrate the tests to a different jury." 4. "I have personally observed my client unable to walk from one side of my office to the other many times."

This case is a witch hunt out for innocent blood and you must find my client not-guilty.

A prosecutor is assigned a high-profile drug trafficking case She is told to prosecute one of the passengers in the car for dealing drugs. The defense attorney representing the passenger talks to the press and says the case is an "embarrassment" to the District Attorney's office because they have "no evidence" and the entire proceeding is a "cover-up of the fact the police planted evidence at the scene." The prosecutor believes the defense attorney is trying to poison the jury pool. She is planning on calling her own press conference at which she will tell the reporters that the defendant admitted to the offense in open court during a pre-trial suppression hearing when confronted with evidence his fingerprints were inside the baggies of the drugs seized in the car. Which of the following is correct? 1. The defense attorney may make allegations to the press that the proceeding is a cover-up and the police planted evidence at the scene. 2. The prosecutor's proposed statements at the press conference are proper if a reasonable attorney would believe they are necessary to protect the state from the prejudicial affect of the defense attorney's speech and are limited as is necessary to mitigate the recent adverse publicity. 3. The lawyer shall not make the proposed statements but she must tell the Bar about the defense attorney's statements or she will be subject to discipline. 4. A prosecutor may not make any statement that has a substantial likelihood of heightening public condemnation of the accused

The prosecutor's proposed statements at the press conference are proper if a reasonable attorney would believe they are necessary to protect the state from the prejudicial affect of the defense attorney's speech and are limited as is necessary to mitigate the recent adverse publicity.

Teresa and Whitney are licensed attorneys who have both taken off a few years to stay at home with their small children. After meeting one day at Chick-fila, the moms decide to form a partnership that will allow each mom to work part-time practicing trusts and estates law. The decide to incorporate to avoid malpractice liability, structure the corporation such that their children are conveyed an interest and appoint a fiduciary representative who, in the unlikely event that either of them dies, will hold stock in the corporation for a reasonable time during the administration and ultimately pass the corporate stock to their children. As it has been some time since the attorneys have practiced law, which of their plans are proper? 1. They may avoid malpractice liability by incorporating. 2. They may incorporate and appoint a fiduciary representative who, in the unlikely event that either of them dies, will hold stock in the corporation for a reasonable time during the administration and ultimately pass the corporate stock to their children. 3. They may incorporate, convey an interest to their children and appoint a fiduciary representative who, in the unlikely event that either of them dies, will hold stock in the corporation for a reasonable time during the administration. 4. They may incorporate and appoint a fiduciary representative who, in the unlikely event that either of them dies, will hold stock in the corporation for a reasonable time during the administration

They may incorporate and appoint a fiduciary representative who, in the unlikely event that either of them dies, will hold stock in the corporation for a reasonable time during the administration

Teresa and Whitney are licensed attorneys who have both taken off a few years to stay at home with their small children. After meeting one day at Chick-fila, the moms decide to form a partnership that will allow each mom to work part-time practicing trusts and estates law. The decide to incorporate to avoid malpractice liability, structure the corporation such that their children are conveyed an interest and appoint a fiduciary representative who, in the unlikely event that either of them dies, will hold stock in the corporation for a reasonable time during the administration and ultimately pass the corporate stock to their children. Are their plans proper? 1. They may avoid malpractice liability by incorporating. 2. They may incorporate and appoint a fiduciary representative who, in the unlikely event that either of them dies, will hold stock in the corporation for a reasonable time during the administration. 3. They may incorporate and appoint a fiduciary representative who, in the unlikely event that either of them dies, will hold stock in the corporation for a reasonable time during the administration and ultimately pass the corporate stock to their children. 4. They may incorporate, convey an interest to their children and appoint a fiduciary representative who, in the unlikely event that either of them dies, will hold stock in the corporation for a reasonable time during the administration.

They may incorporate and appoint a fiduciary representative who, in the unlikely event that either of them dies, will hold stock in the corporation for a reasonable time during the administration.

A doctor hired a lawyer to represent her in a complex medical malpractice case. The lawyer interviewed expert witnesses and sent summary motions to the plaintiff's attorney. However, not even half way through the trial schedule, the lawyer's child was run over and brutally killed by an intoxicated driver while the child was playing ultimate Frisbee at a public park. The lawyer became severely depressed and stopped getting out of bed. The lawyer called the doctor and told her she needed to find another attorney to take the case. The lawyer recommended several other attorneys in town with excellent reputations. However, the client refused to allow the lawyer to turn her files over to another attorney as she believed that her lawyer was the best in town. What should the lawyer do? 1. Turn the files over to any one of the other competent attorneys in town. 2. Withdraw and send the files to the client. 3. Set the case aside, try to get recover and began working on the case again as soon as possible. 4. Withdraw after explaining to the judge the situation and obtaining the permission of the tribunal.

Withdraw after explaining to the judge the situation and obtaining the permission of the tribunal.

A trusts and estates attorney, Rob, wrote a will for a wealthy client that left all his money to his dog. The client died a few years later and his heirs contested the testamentary capacity of the client and the authenticity of the will. Although Rob's partner knew that Rob would have to testify as to the authenticity of the will, thepartnerstill agreed to represent the estate. He plans to call Rob to the stand at the trial to testify that the will is authentic. Is the conduct proper? 1. No, Rob may not testify about the authenticity of the will. 2. No, Rob must testify about the authenticity of the will but that precludes his law partner from representing the estate. 3. No, unless prohibiting the law partner from representing the estate would work a substantial hardship on the client. 4. Yes, Rob may testify about the authenticity of the will and his partner may represent the estate assuming it is not a conflict of interest.

Yes, Rob may testify about the authenticity of the will and his partner may represent the estate assuming it is not a conflict of interest.

Attorney Nerd is chatting online with her Star Wars fan group on May the 4th Day about whether Rey should have been Kylo Ren's sister instead of love interest. The conversation takes an unexpected turn when another one of the fans shares that she "wishes" Rey had been Kylo Ren's sister because her brother beats her up and it would inspire her to stand up to him more often. Concerned that someone is being abused, Attorney Nerd inquires further and realizes the user is a disabled adult and her adult brother is supposed to be her caretaker. Attorney Nerd posts on the discussion board, "I don't know if you know this, but I am an attorney. I definitely think you should get help and probably consult with an attorney. I'm available if you'd like a consultation at my office." Is Attorney Nerd subject to discipline under the Model Rules? 1. Yes, a chat room is a real-time electronic communication. 2. Yes, because the communication was solicitation which is strictly regulated under the ABA Model Rules. 3. No, because the communication is advertising that is neither false or misleading. 4. No, under these exigent circumstances the attorney had to make the user aware of her rights to bring an end to an abusive situation

Yes, a chat room is a real-time electronic communication

Ariel, a law clerk under Supreme Court Justice Alito, is approached by the ACLU as a potential new employee. Ariel is currently working on a discrimination case for Justice Alito where the ACLU represents the plaintiff. Ariel informs Justice Alito that she is entering contract negotiations for employment with the ACLU. Are the law clerk's actions proper? 1. No, unless all parties to the proceeding give informed consent in writing. 2. No, unless Justice Alito agrees to properly screen the law clerk. 3. Yes, a law clerk may negotiate employment with a party in a matter in which they have substantially participated if they notify the judge. 4. Yes, a law clerk may negotiate employment with a party in a matter in which they have substantially participated.

Yes, a law clerk may negotiate employment with a party in a matter in which they have substantially participated if they notify the judge.

Scrooge, who recently lost a lawsuit brought by Bob Cratchit for withheld wages, informs his attorney that he wishes to appeal the case. The attorney believes there is no hope that an appeals court would reverse the damages considering his client's reputation in the community; however, with a different client he might win the appeal. Still, Scrooge tells the attorney that he has run the numbers, and considering the paltry amount he is paying the attorney, it is in Scrooge's financial interest to delay execution of the judgement for as long as possible. After examining the benefits, the attorney agrees to exhaust all appeals. Is the attorney subject to discipline? 1. Yes, the appeal is frivolous. 2. Yes, because Scrooge's finances are not an interest of the client that the law recognizes to postpone a judgement. 3. No, because even if the attorney believes the appeal will ultimately fail, it is not frivolous if there is still a basis in law and fact for success. 4. No, because even though the lawyer may decide strategic matters in certain circumstances, the client controls all major decisions in the litigation.

Yes, because Scrooge's finances are not an interest of the client that the law recognizes to postpone a judgement.

An attorney represented a wealthy family for years with real estate closings and wills. When their small child died in a sudden, terrible car accident the attorney went to visit the family to pay his respects. The husband was very drunk when he arrived and told the attorney he could not live with the guilt since he had been the one driving when the accident occurred and he planned to commit suicide that evening. He told the attorney this in confidence and asked him to make sure his money went to his other children and wife. As soon as the attorney left the house he called the police and alerted them that the husband was suicidal. The husband was taken to a psychiatric hospital and put on suicide watch. Was the attorney's conduct proper? 1. Yes, because his client was drunk. 2. Yes, because an attorney may reveal confidential information when a client proposes to act in a way that will result in death or substantial bodily harm. 3. No, he violated his ethical duty by revealing his client's confidences. 4. No, he was told to make sure the money went to the children and he didn't.

Yes, because an attorney may reveal confidential information when a client proposes to act in a way that will result in death or substantial bodily harm

An attorney who has a bankruptcy practice and has made a big name for himself in the specialty learns that his brother is about to declare bankruptcy. He calls his brother and tells him he will represent him for free. However, his brother is embarrassed that his successful sibling even knows about his financial difficulties and tells him, "I don't want your help. Leave me alone! Never talk with me about this again!" and hangs up the phone. Knowing his brother needs his help, the attorney calls his sister and asks her to urge his brother to let him take the case. He waits a few days and then calls his brother again, urging him to let him take the case. Is the attorney subject to discipline? 1. Yes, because he called and attempted to solicit a client. 2. Yes, because he called his brother after his brother made his desire not to be solicited clear. 3. No, because you are allowed to solicit close family members. 4. No, because his sister urged her brother to let him take the case and not the attorney.

Yes, because he called his brother after his brother made his desire not to be solicited clear

An attorney works on Redstone Arsenal for the Missile Defense Agency (MDA) reviewing federal contracts between MDA and local contractors. He never has to litigate, mediate or arbitrate before any tribunal as his job is simply to ensure the labyrinth of federal regulations on government contracts are properly followed. He is licensed to practice in Washington D.C. but was transferred to Huntsville, Alabama because the agency could pay him less salary in Huntsville than in Washington, DC on the same GS pay scale when adjusted for the cost of living. The attorney resented the move and refused to take the Alabama Bar Exam. In addition to reviewing contracts for MDA, he has also recently taken to doing a little real estate contract review, for his fellow federal MDA employees only, at his home in Madison to earn some extra cash. Is the lawyer subject to discipline for unauthorized practice of law? 1. No, because the arsenal is federal property and Alabama has no jurisdiction. 2. No, because all his practice is limited to federal law and federal employees. 3. Yes, because he cannot review real estate contracts in Alabama. 4. Yes, because he cannot review contracts in Alabama.

Yes, because he cannot review real estate contracts in Alabama.

A law firm represents a successful businessman who has given the firm almost all of the legal work that arose in his business. The businessman is being sued by a competitor and stands to lose his entire fortune. The attorney for the plaintiff has filed a motion regarding the application of a non-compete clause from decades ago that, if granted, would probably result in the businessman losing the case. The motion has little merit and shouldn't be granted. The named partners call in a recently hired associate. The associate clerked for the judge and wrote most of her re-election campaign speeches. The partners tell their client it would be to his advantage to have the new associate argue the motion in front of the judge. The associate tells the businessman, "Don't worry—Judge Smith owes me a favor." Is the Associate subject to discipline? 1. Yes, because he is using his knowledge of the judge and the respect the judge has for him to gain an advantage. 2. Yes, because he is verbally implying the judge will give him preferential treatment. 3. No, because he did not expressly say that the judge would rule in his favor. 4. No, because the motion is meritless he is not exercising improper influence.

Yes, because he is verbally implying the judge will give him preferential treatment.

Alfred had trouble finding clients for his estate planning practice. Occasionally, he volunteered at the "Wills for Heroes" voluntary lawyer program to meet potential customers and do some pro bono work. The standard interview question paperwork provided to the police and fireman clients included a provision that attorneys were not liable for malpractice because the legal services were provided free of charge. Alfred never looked at the interview paperwork, as he always preferred to thoroughly interview his clients in person, and did not see the provision. The clients kept the paperwork. One policeman he interviewed had an estate worth over five million dollars. Alfred informed him that he was beyond the income limits of the volunteer lawyer program. The policeman asked him if he "did wills" and if he would do his will? Alfred offered to do the will for $1,000 and told him to come to his office to discuss terms. Alfred had the policeman sign a contract that included an arbitration clause he explained in detail. Alfred set up a living will for the policemen that put all his assets in a trust. After a few years, the policeman realized the living will had to be continually updated and required him to pay more and more money to the attorney. Frustrated, he accused the attorney of misleading him about the true cost of the will and threatened to sue. Alfred knew he couldn't afford a frustrated client in his fledgling practice—so he offered to settle with the policeman by writing a standard will for him that would not need updating at no additional cost. The policeman easily agreed to the proposal and signed a contract that he would not sue the attorney or file a State Bar complaint against him. Alfred never mentioned whether the policeman should consult outside counsel since the policeman's complaints were completely baseless and he was basically doing him a favor to maintain his reputation among the police force. Is the attorney subject to discipline? 1. Yes, because he prospectively limited his malpractice liability and settled with the client. 2. Yes, because he settled with the client without advising him to get outside counsel. 3. Yes, because the interview paperwork limited his liability, the arbitration clause limited his liability and he failed to advise his client to obtain outside counsel before signing the settlement. 4. No, the arbitration clause is perfectly legal and Alfred never even saw the interview paperwork provided to the client by the volunteer lawyer program.

Yes, because he settled with the client without advising him to get outside counsel

A young lawyer was hired right out of law school to practice as in-house counsel for a teacher union. The lawyer mainly handles contracts and negotiations and has never set foot in a courtroom. One of the teachers the union represents is accused of sexually molesting three of her pupils and is charged with a felony. After thoroughly investigating the case, the lawyer and his union believe the teacher is innocent and the lawyer offers to represent the teacher for free. A few days before trial, the lawyer realizes his trial skills are abysmal and he is simply not competent to defend the teacher. He asks the trial judge for permission to withdraw. The judge thoroughly questions the lawyer and determines that the lawyer is very anxious but still competent. The judge denies the motion to withdraw but postpones the trial for a week to give the young attorney time for more preparation. Instead of doing what the judge ordered, the lawyer advises the teacher that he cannot represent her and that she should find other counsel. He gives her all the files in the case and tells her she is better off with someone with more experience. Is the lawyer subject to discipline? 1. Yes, because he told the teacher he would not represent her and withdrew. 2. Yes, because he was never competent to handle the case and waited too late to attempt to withdraw. 3. No, because he was not competent to represent the client and that is good cause to withdraw. 4. No, because he took the case pro bono.

Yes, because he told the teacher he would not represent her and withdrew.

A tax attorney files and reviews taxes for a major international corporation. In order to get the taxes done on time, the attorney allows his secretary to see all the confidential merger information so she can check his math. After years of representing the corporation, the tax attorney discovers that the corporation has been funneling false paperwork through him to lower their effective tax rate. The company has been able to evade millions of dollars in taxes. The attorney withdrew as soon as he learned of their shenanigans and reported the corporation to the IRS. Is the attorney's conduct proper? 1. No, the information involves past behavior of his client and is therefore confidential and should not have been reported. 2. No, his secretary should not have been allowed to see the confidential merger information. 3. Yes, because the company has committed a crime. 4. Yes, because reporting would rectify substantial financial injury and the corporation used the attorney's services to commit the fraud

Yes, because reporting would rectify substantial financial injury and the corporation used the attorney's services to commit the fraud

Despite a direct question that should have elicited the information, an applicant for admission to law school does not reveal that, prior to attending the University of Alabama and attaining a 4.0 GPA, she failed out of a parochial college. The applicant "doesn't think it would be fair" to count it against her since she is now 28 years old and the year in parochial college occurred almost a decade ago. After admission to law school, as a third year law student, she applies for admission to the bar. The state bar application includes the question, "Are there any material misstatements or omissions on your law school application?" She does not disclose the omission. She also does not disclose a class C conviction for Minor in Possession of Alcohol that also occurred in her "wild" days. The applicant is admitted to the bar. Is she subject to discipline? 1. Yes, because even a conviction of a traffic infraction must be disclosed to the bar. 2. Yes, as the failure to disclose the material omission on the law school admission but not for the Minor in Possession charge as it is not a crime of moral turpitude. 3. Yes, because she failed to disclose the fact she made material omissions on her law school application and she knowingly fail to respond to a lawful demand for information. 4. No, because the lawyer was not admitted to the bar at the time the admissions application was completed.

Yes, because she failed to disclose the fact she made material omissions on her law school application and she knowingly fail to respond to a lawful demand for information.

A young prosecutor works in municipal court. The judge does not want to appoint attorneys for the misdemeanor cases because the city budget is under-funded and barely functioning. He asks the prosecutor to try to work a "good deal" out with the defendants and if they cannot agree he will appoint someone. The prosecutor agrees to only meet with those defendants whose cases the prosecutor is willing to plead down to minor traffic infractions. The judge agrees that those clients who the prosecutor believes need to plea to higher charges the judge will appoint counsel to represent. Part of the required forms the prosecutor must explain to the Defendant to reduce the charge to a traffic offense is a jury waiver. Is the prosecutor subject to discipline? 1. Yes, because a prosecutor cannot meet directly with a defendant. 2. Yes, because the prosecutor sought a jury waiver. 3. No, unless the prosecutor knew or reasonably should have known that the defendant's misunderstood the prosecutor's role in the matter. 4. No, because the defendants have a right to represent themselves pro se.

Yes, because the prosecutor sought a jury waiver.

A prosecutor was gathering evidence which she hoped to use to convict the wife of the mass murderer in Orlando as a co-conspirator. As it turned out, the wife had been represented by a defense attorney for shop-lifting at a gun show. The prosecutor had good reason to believe that the defense lawyer had information in his files that was essential and non-privileged regarding the wife's aid in compiling an arsenal for the planned attack. The prosecutor could feasibly contact all 50 vendors at the gun show to compile a list of stolen goods that the wife may have stolen without observation; but, to save public money and time the prosecutor arranged for a subpoena to be issued to the defense lawyer for the evidence. Is the prosecutor subject to discipline? 1. Yes, because there was a feasible alternative way to obtain the information. 2. Yes, because prosecutors may not subpoena defense lawyers to obtain evidence about the defense lawyers' present or former clients. 3. No, because the evidence sought by the prosecutor was not privileged. 4. No, if the evidence sought by the prosecutor was essential to the successful completion of the prosecution.

Yes, because there was a feasible alternative way to obtain the information

A recent law graduate is appointed on a shoplifting charge. The defendant was not arrested at the scene but was identified by a loss prevention officer at a later date. The defendant tells the lawyer that he is innocent. In fact, his girlfriend is prepared to testify that the defendant was helping her babysit her siblings at her house at the time the shoplifting occurred. The lawyer interviews the defendant's girlfriend and she repeats the alibi. However, the lawyer has a mighty suspicion that the girlfriend is lying to protect her boyfriend. In fact, the prosecution's case is fairly strong that the defendant was the perpetrator and the lawyer's intuition is that the girlfriend's testimony would be perjury. The lawyer warns them both about the dangers of perjury but they both want to testify. May the lawyer proffer the testimony to the court of the defendant, his girlfriend, or both? 1. Yes, as to the defendant but not his girlfriend. 2. Yes, as to the girlfriend but not the defendant. 3. No, neither can testify. 4. Yes, both may testify.

Yes, both may testify

An attorney interned for a medical malpractice firm. He helped with a case involving Martha May who was severely injured during the birth of her first child and as a result suffers from long-term pain that can only be managed by high dosages of opiates. Five years later, the intern is now an attorney and opens his own personal injury firm. One of his first big cases to walk thought the door involves a terrible accident that claimed the life of a small child. The plaintiff has a great case—partly because Martha May is the defendant and the attorney knows the opiates she takes for her condition likely had a significant role in the accident. Is it proper for the attorney to take the plaintiff's case against Martha May? 1. Yes, but only if he fully discloses his relationship to Martha May to his new client. 2. Yes, but only if Martha May and the plaintiff give written, informed consent. 3. Yes, because he was only an intern and therefore does not owe Martha May any fiduciary duties. 4. No, unless the confidential information would be easily discovered in the course of the proceedings.

Yes, but only if Martha May and the plaintiff give written, informed consent

A few days after giving birth, Lynn, a Chinese foreign student at the University of Alabama got in a fistfight with her mother-in-law. Her husband, also from China, called the police to mediate the fight. When the police arrived they had trouble understanding the broken English of the family and ended up arresting Lynn for domestic violence even though Lynn had not initiated the fight because Lynn remained quiet during questioning. Lynn's husband and mother-in-law both begged the police not to arrest her at the scene; but, the police were worried about liability were something to happen after they left if they failed to separate the parties. Lynn will lose her scholarship and be unable to return to the United States if convicted. Janice, an attorney, met with the family a few days after the arrest. The mother-in-law has been subpoenaed to come to court but her return flight to China is before the court date. No one in the family has been contacted by the state yet. Janice informs the mother-in-law that she should not talk to the prosecutor. Moreover, she tells the mother-in-law that leaving the country is a good idea and will "take care" of the whole mess. Did the attorney violate the ABA Rules? 1. No, the mother-in-law is a relative and it is not against her interest to be uncooperative or to leave the country. 2. No, the defense attorney may communicate with the victim. 3. Yes, she advised a victim that she could refrain from speaking with the prosecutor and encouraged her to leave the country. 4. Yes, even if she reasonably believes that the mother in law's interests will not be adversely affected by not talking with the prosecutor, she still encouraged a witness to leave the country

Yes, even if she reasonably believes that the mother in law's interests will not be adversely affected by not talking with the prosecutor, she still encouraged a witness to leave the country

A real estate attorney assists a client in purchasing an abandoned mall. The client intends to invest millions of dollars in updating the mall and converting it into a call center for a large cell phone company. Knowing that the client's plans will revitalize the area and increase the value of surrounding properties, the attorney uses the confidential information and buys a struggling Taco Bell franchise close to the abandoned mall. As expected, the call center brings hundreds of employees to the area and the Taco Bell makes bank. Is the lawyer subject to liability? 1. No, the Taco Bell does not disadvantage her client. 2. No, as long as she does not tell people her client's plans she can use the confidential information. 3. Yes, even though the information is not to the disadvantage of her client, she owes the client a fiduciary duty which means she is civilly liable and any profit from the Taco Bell transaction belongs to the client. 4. Yes, the lawyer needs her client's informed consent to use the confidential information in any scenario.

Yes, even though the information is not to the disadvantage of her client, she owes the client a fiduciary duty which means she is civilly liable and any profit from the Taco Bell transaction belongs to the client.

In the midst of the Enron scandal, an executive is caught red-handed destroying reams of paperwork documenting his embezzlement of investor funds. The executive meets with his long-time friend and attorney to beg for his help in getting out of the mess. He asks, "What would happen if I just skipped town before the grand jury can get back with an indictment?" The attorney began explaining the consequences of skipping out of bail, that it is illegal, and the fact that most countries have extradition agreements with the United States. However, as he talked he Googled the search terms "countries without extradition agreements" and "Russia" and turned his computer screen such that the executive could see it. Is the lawyer subject to discipline? 1. Yes, he assisted his client in criminal conduct. 2. Yes, he should not have assisted the client by discussing extradition agreements and bail jumping much less looked up what countries he could escape to without facing extradition. 3. No, he had a right to explain the consequences of a proposed course of conduct with the client. 4. No, he told the client his proposed course of conduct was illegal and he has no duty to control his client's behavior.

Yes, he assisted his client in criminal conduct.

A state supreme court judge is married to a successful real estate agent for Coldwell-Banker. Every year the real estate company ran a lottery (with every home sold worth another entry) for an all-expense-paid trip to Hawaii for two. The judge's spouse won the lottery and the two took a fabulous vacation to Hawaii. The judge did not report the vacation. Coldwell-Banker has appeared before the state supreme court on cases in the past. Was the judge's conduct proper? 1. Yes, her husband won the prize. 2. Yes, there is no appearance of impartiality. 3. No, Coldwell-Banker could appear before the court and the gift would appear to undermine impartiality. 4. No, the judge could benefit from the prize but she still must report it.

Yes, her husband won the prize

A defense attorney with a high-volume practice before a judge sends her a $500 gift card to Dillard's as an Anniversary gift. The defense attorney was a bridesmaid in the judge's wedding in law school. The judge accepts the gift and duly publicly reports it. Are the judge's actions proper? 1. No, because the attorney appears before the judge. 2. No, the gift is not a campaign contribution and is thus disallowed. 3. No, because the value of the gift exceeded $200. 4. Yes, if $500 doesn't reasonably undermine impartiality because the judge publicly reported the gift.

Yes, if $500 doesn't reasonably undermine impartiality because the judge publicly reported the gift

Lawyer Lynn was a home health provider before she became a lawyer. A few years after passing the bar, she ran into one of her former home health clients when visiting her grandmother at an assisted care facility. The elderly woman appeared to be in an advance state of dementia. She no longer knew her name or where she lived. Lynn could tell she was not getting the care she needed considering her condition. She called the woman's daughter—as she still had her number in her cell phone. The daughter confirmed that the woman was not getting the care she needed. She also told Lynn that her brother had taken control of the check book and was using his mother's substantial assets to buy himself cars, boats and pay off his mortgage. Lynn explained to the woman's daughter that the mother needed someone to legally intervene. The daughter said she didn't know any lawyers. Lynn offered to seek an emergency appointment of a guardian ad litem while the daughter could look for a trusts and estates attorney to take the case. If Lynn acts on the woman's behalf will she be subject to discipline? 1. Yes, if she sends the daughter a bill for her work. 2. Yes, a lawyer cannot act on behalf of a non-client. 3. No, an attorney may act on behalf of a non-client in an emergency where the person with diminished capacity faces imminent, irreparable harm. 4. No, as a former client in-person solicitation is acceptable.

Yes, if she sends the daughter a bill for her work

The in-house counsel for a large petrochemical company in Port Arthur, Texas learns that the company is circulating a private memo around that any repairs to equipment should be "delayed" for a few months until "the price of oil goes up again." The attorney knows that much of the plant was built in the early 1980s and failing to keep equipment repaired is reasonably certain to lead to lethal accidents and costly lawsuits that could even bankrupt the company. The attorney would like to report the company to federal government regulators. May the attorney report his client? 1. Yes, if the attorney first takes the problem to the board of directors and they fail to act. 2. Yes, the attorney is duty bound to protect the employees of the company from the risk of harm. 3. No, the attorney is bound by the duty of confidentiality and the risk of harm is not imminent. 4. No, because the attorney ultimately represents the company and not its constituents.

Yes, if the attorney first takes the problem to the board of directors and they fail to act.

Lawyer Johnny John specializes in personal injury cases. Lately, the lawyer has won big verdicts against Johnson & Johnson because of their defective manufacturing of the Acetabuler and ASR hip resurfacing systems, manufactured by Johnson & Johnson subsidiaries, DePuy Orthopedics. The metal on metal friction is known to cause severe infections, fractures, loss of bone and excruciating groin pain to its users, leading at times to complete immobility. It was serious enough to lead FDA to issue a warning. However, many people with the implants have never been told the implant was defective. The lawyer creates a commercial explaining the defective product and also featuring many of his former clients holding oversized cardboard checks that show large dollar amounts. The clients all say, "Thanks Johnny!" Is the lawyer subject to discipline? 1. Yes, unless the commercial includes a disclaimer that the results for these clients might not be typical and every case must be evaluated on its own merits. 2. Yes, if the commercial would cause a reasonable person to form an unjustified expectation of similar results. 3. No, advertisement is protected by the First Amendment as commercial speech even if it is distastefully done as long as it is truthful. 4. No, but only if the amount of proceeds on the checks accurately reports what portion of the proceeds went to the client and what portion was retained by the lawyer.

Yes, if the commercial would cause a reasonable person to form an unjustified expectation of similar results.

A famous defense attorney takes a DUI breath test case knowing that the key to winning will be destroying the credibility of the state's excellent expert witness. Using expensive private investigators he finds out everything he can about the expert including privileged information in her personnel file. At trial, he cross-examines the witness on her expertise and understanding of the breath testing instrument as well as her education and awards. He points out the expert has never published any scientific papers. He asks accusingly if it is true that she once worked under a supervisor at the CSI lab who fabricated evidence. Finally, he asks the expert, "Isn't it true that although you claim to be a scientist you made a D in high school Chemistry?" Is the defense attorney subject to discipline? 1. No, because embarrassing the witness is a viable trial technique. 2. No, all of his questions were designed with the substantial purpose of destroying the credibility of the state's key witness and sometimes that causes embarrassment. 3. Yes, all his questions were designed solely to embarrass, degrade, or humiliate the witness. 4. Yes, if the private investigators used methods of obtaining evidence that violated the legal rights of the expert

Yes, if the private investigators used methods of obtaining evidence that violated the legal rights of the expert

Melissa, a mom in Huntsville who hasn't been in the work-force for years, decides to go to law school. Knowing her grades in college were not great, she tries to find someone to write her letter of recommendation to the law school that will be impressive. She has one friend whose father is a judge in Athens. She asks the friend if her father will write a letter of recommendation for her to get into law school. The judge agrees to write the letter. He uses his judicial letterhead to pen a short letter that simply says he has known Melissa since she was a teen and she is tenacious and would be an excellent asset to the law school. Were the judge's actions proper? 1. Yes, if there is no likelihood the judge is attempting to exert pressure on the law school. 2. Yes, references and recommendations are allowed on judicial letterhead. 3. No, the judge is abusing the prestige of the office by writing the letter on judicial letterhead. 4. No, the judge is allowing Melissa to abuse the prestige of the office to advance her own interests.

Yes, if there is no likelihood the judge is attempting to exert pressure on the law school.

Being the only lawyer in town, Halley takes out a small ad in the phone book that reads: "Elkton Town Legal Clinic... operated by Halley, your hometown private attorney." Has Halley's advertisement violated the Model Rules? 1. No, she is not subject to discipline because advertisements in phone books are specifically allowed in the Model Rules. 2. No, she is not subject to discipline because this is advertisement and not solicitation and the First Amendment protects Commercial Speech. 3. Yes, she may be subject to discipline for making a false statement. 4. Yes, she may be subject to discipline for failing to include a disclaimer that her firm is not connected with a government agency or public or charitable legal service organization.

Yes, she may be subject to discipline for failing to include a disclaimer that her firm is not connected with a government agency or public or charitable legal service organization.

In the midst of a hotly contested trial, an issue arose regarding the admissibility of certain hearsay evidence. As it was close to 5:00 pm, the judge sent the jury home and told the attorneys to research the issue and present their arguments in the morning. Attorney May was an excellent researcher and after arduous research found the controlling case in their jurisdiction. Unfortunately, it went against her case. Knowing that her opposing counsel had his daughter's dance recital that night, she correctly surmised he would not have the time to find the case. The next morning, Attorney May cited the case as one of the cases she had found controlling in their jurisdiction and provided the court with a copy of it along with a stack of other cases. However, she failed to inform the court what the holding of the case was or that it went against her argument. Is the attorney subject to discipline? 1. Yes, because the attorney has a duty to objectively explain cases to the court. 2. Yes, she mislead the tribunal. 3. No, it is her duty to present the cases in the light most favorable to her client. 4. No, the attorney is in an adversarial position and she gave a copy of the case to the judge.

Yes, she mislead the tribunal.

A real estate attorney assists a client in purchasing an abandoned mall. The client intends to invest millions of dollars in updating the mall and converting it into a call center for a large cell phone company. Knowing that the client's plans will revitalize the area and increase the value of surrounding properties, the attorney uses the confidential information and buys a struggling Taco Bell franchise close to the abandoned mall. As expected, the call center brings hundreds of employees to the area and the Taco Bell makes bank. Is the lawyer's conduct proper under the ABA Rules? 1. Yes, the Taco Bell does not disadvantage her client. 2. Yes, as long as she does not tell people her client's plans she can use the confidential information. 3. No, even though the information is not to the disadvantage of her client, she owes the client a fiduciary duty which means any profit from the Taco Bell transaction belongs to the client. 4. No, the lawyer needs her client's informed consent to use the confidential information in any scenario.

Yes, the Taco Bell does not disadvantage her client.

A lawyer is approached by parents who recently lost their child in an accident involving an alligator at Disney World. The parents are furious that the resort (where they were paying nearly $900 per night to stay) failed to adequately inspect their property and erect safeguards to keep dangerous animals out or warn them that such animals might be on the property. The father went to law school for one year and has some legal experience. He asks the lawyer to file a premise liability case against Disney alleging negligence, gross negligence, assault and seeking punitive damages. After doing a little preliminary research and discovery, the lawyer comes to the conclusion that there is no basis in law or fact for the assault charge. The gross negligence charge will almost certainly be dismissed; but, the plain vanilla negligence charges for an invitee might stick. He informs the client that they should drop gross negligence, assault and punitive damages from their initial complaint and proceed on the negligence charges alone. However, the father completely disagrees with the lawyer's analysis and insists on the charges he decided were appropriate from his old law school notes. The lawyer obeys his client and files the complaint. 1. No, because even though the lawyer may decide strategic matters, the client controls all major decisions in the litigation and the lawyer must include all the claims the father directs him to include. 2. No, individual frivolous issues may be asserted in the initial complaint as long as the case itself is not frivolous. 3. Yes, the assault allegation is frivolous. 4. Yes, the assault and gross negligence allegations are frivolous

Yes, the assault allegation is frivolous

An employee of a large automobile corporation wanders into the break room and sees one of the attorneys who works for the general counsel's office. The employee approaches the attorney and guiltily begins to explain that he is having trouble sleeping because he is fairly certain that he knows what is causing the ignition malfunction that has caused a series of accident fatalities. The attorney tells the employee to tell him "everything." The employee admits that his group at work identified the defect a few years ago but were afraid they would be fired because the defective ignition switch had already been installed in a large number of cars--so they covered up their mistake and falsified testing data on the component. The attorney goes straight back to his desk and emails the CEO and the COO the employees name and the details of their conversation. Is the attorney subject to discipline? 1. No, the attorney represents the corporation not the employee. 2. No, the employee did not indicate that the information was confidential. 3. Yes, the attorney broke client confidentiality with the employee and failed to warn him that the attorney represents the corporation's interests not his. 4. Yes, the attorney failed to warn the employee that the attorney represents the corporation and not his interests.

Yes, the attorney broke client confidentiality with the employee and failed to warn him that the attorney represents the corporation's interests not his.

After a long and arduous jury trial, the prosecutors and defense attorneys who worked on the case decide to go get a drink together. As luck would have it, a local judge was holding a campaign fundraising event with an open bar close to the courthouse. After one too many alcoholic beverages, one of the defense attorneys leaned over to the lead prosecutor and said, "Your expert witness was pretty fancy. Does he usually wear pants or a skirt to work?" Is the defense attorney subject to discipline? 1. No, the First Amendment protects the comments by the defense attorney. 2. No, the comments are biased but they are not covered by the ABA Model Rules. 3. No, the comment was not directed to the expert witness and thus are not harmful. 4. Yes, the comments were demeaning verbal conduct.

Yes, the comments were demeaning verbal conduct

A client hired a lawyer to draft a will for him. The client willed his entire estate to a 19-year-old orphan named Jarrod. The client told the lawyer in confidence that he was neither a relative nor friend of the boy—but, he felt responsible for him because he murdered his entire family. One day after signing the will, the client committed suicide. In due course, all assets were given to the boy and the probate court closed the estate. The lawyer never told anyone that the client confessed to the murder. A few years later, a young prosecutor tried an innocent man for the "cold case" murder and he was given the death penalty. By some dark magic, the appeals are expedited for the death penalty case and the US Supreme Court denies all the man's appeals and the execution is set for next week. May the lawyer voluntarily tell the innocent man's defense counsel what his client told him in confidence about killing the family? 1. Yes, the lawyer not only may, but he must, tell the defense counsel what the client told him. 2. Yes, the lawyer may tell, but he would not be subject to discipline if he decided not to reveal the confidence. 3. No, the lawyer would be subject to discipline if he told defense counsel because the attorney-client privilege survives the death of the client. 4. No, because the client's confidential confession to the lawyer would be inadmissible heresy if offered against the prosecution in the murder trial.

Yes, the lawyer may tell, but he would not be subject to discipline if he decided not to reveal the confidence.

An attorney got an odd call from his co-worker to meet him at a local bar. When he arrived, the co-worker was sloshed and told him that he had just discovered his wife had cheated on him with multiple partners and as a result he had contracted a STD. The co-worker was furious and yelled several times that he was going to "kill that witch" when he got home and wanted the attorney to get him out of any criminal proceedings since she clearly "had it coming." Everyone in the bar kept glaring at the co-worker and the attorney and whispering about the co-worker's outbursts. The attorney told the co-worker to calm down and get some sleep and he would file the divorce paperwork in the morning. He helped the co-worker get a cab and told him not to do anything stupid and just call him in the morning. Unfortunately, the co-worker was still enraged when he got home and knocked his wife down the stairs—breaking her neck and killing her. The co-worker blamed the attorney for letting him go home in that condition and fired him from the jail. He hired a different attorney to defend him. The prosecutor called the first attorney to testify at trial about what the co-worker had said at the bar to prove it was not an accident. The new attorney objected to the testimony based on the attorney-client privilege. Does it violate attorney-client privilege for him to testify? 1. No, because he no longer represented the co-worker as his attorney. 2. No, because the co-worker blamed him for the murder and it is self-defense. 3. Yes, because the co-worker did not use his services in committing the crime. 4. Yes, unless other people in the bar heard the co-worker's admissions.

Yes, unless other people in the bar heard the co-worker's admissions

Leo practices law in Hollywood, Alabama. He represents Client Carl in a uncontested divorce action against Candace. The parties do not have children and have been able to settle all matters fairly amenable and are just awaiting their day in court for the judge to sign off on their agreement. Leo has known Carl and Candace his whole life and went to high school with them both. Candace, who knows Leo only represents Carl in the divorce and is unrepresented in the divorce matter, comes to his office and tells Carl that she is thinking about suing a local doctor for malpractice since he failed to warn her that taking certain medications could cause her to become infertile. Leo thinks it's a good case but worries suing now would mean Carl might have a right to any settlement offer. He warns Candace that this might be a problem if he took the case. Leo calls Carl and informs him about the entire situation as well. Carl consents to Leo handling the case as does Candace. Is Leo subject to discipline? 1. Yes, it's a conflict of interest to represent clients with adverse interests and consent will not cure the conflict. 2. Yes, unless the consent is in writing. 3. No, the divorce and medical malpractice claims are not adverse so there is not a concurrent conflict of interest. 4. No, he reasonably believes he can provide competent representation, the new client is not suing the old client in the same matter, they consented.

Yes, unless the consent is in writing


Conjuntos de estudio relacionados

Culture and Society FAU Pearson Revel Chapter 11

View Set

Religion, Spirituality, and Nursing Care

View Set

earth science unit 2 study guide

View Set

BUSM 4100 FInal - Project Management

View Set

Computer Concepts Modules 4-5 Test

View Set